Regularisierung des Casimir-Effekts

Lassen Sie mich zunächst sagen, dass, obwohl der Casimir-Effekt Standard-Lehrbuch-Zeug ist, das einzige QFT-Lehrbuch, das ich in Reichweite habe, Weinberg ist und er nicht darüber spricht. Die einzige Quelle, die ich derzeit zu diesem Thema habe, ist Wikipedia . Trotzdem vermute ich, dass diese Frage angemessen ist, da ich mich nicht erinnere, dass sie in anderen Lehrbüchern angesprochen wurde

Naiverweise führt die Berechnung des Casimir-Drucks zu unendlichen Summen und erfordert daher eine Regularisierung. Es können mehrere Regler verwendet werden, die die gleiche Antwort liefern: Zeta-Funktion, Wärmekern, Gauß, wahrscheinlich auch andere. Die Frage ist:

Was ist der mathematische Grund, warum alle Regulierungsbehörden die gleiche Antwort liefern?

Physikalisch bedeutet dies, dass der Effekt unempfindlich gegenüber der detaillierten Physik der UV-Grenze ist, die in einer realistischen Situation mit den Eigenschaften der verwendeten Leiter zusammenhängt. Die Wikipedia erwähnt, dass der Effekt für einige kompliziertere Geometrien empfindlich auf den Cutoff reagiert, warum also für das klassische Beispiel paralleler Ebenen nicht?

BEARBEITEN: Aaron lieferte einen wunderbaren Terry Tao-Ref, der für diese Ausgabe relevant ist. Aus diesem Text geht hervor, dass die divergierende Summe für Vakuumenergie in einen endlichen und einen unendlichen Teil zerlegt werden kann und dass der endliche Teil nicht von der Wahl des Reglers abhängt. Allerdings das UnendlicheTeil hängt von der Wahl des Reglers ab (siehe Gl. 15 in Taos Text). Jetzt haben wir einen weiteren Parameter im Problem: den Abstand zwischen den Leiterebenen L. Was wir zeigen müssen, ist, dass der unendliche Teil nicht von L abhängt. Dies erscheint immer noch wie ein Wunder, da es für alle Regler passieren sollte. Außerdem funktioniert es nicht für das Spielzeugbeispiel eines masselosen Skalars in 2D, es sei denn, ich bin verwirrt. Für dieses Beispiel sind alle Terme in der Vakuumenergiesumme proportional zu 1/L, daher ist der unendliche Teil der Summenasymptotik auch proportional zu 1/L. Wir haben also ein „Wunder“, das nur für bestimmte Geometrien und Abmessungen geschieht

Ich denke, die Antwort auf die Frage lautet, dass die Antwort für renormalisierbare Theorien unabhängig von der Regularisierung sein muss (sobald Sie die Grenze / Regularisierung weggeschickt haben).

Antworten (5)

Schauen Sie sich hier Terry Taos brillanten Beitrag zur Regularisierung von Zeta-Funktionen an (ich habe das Thema nie verstanden, bis ich diesen Beitrag gelesen habe). Die kurze Antwort ist, dass sie alle dasselbe berechnen, die (geeignet definierte) Asymptotik der divergenten Summe.

Danke Aaron, ein toller Schiedsrichter! Ich glaube, da ist noch etwas unklar. Ich werde die Frage entsprechend bearbeiten
Tao ist großartig, aber das ist einfach keine gültige Antwort auf die ursprüngliche Frage. Taos Text ist so ziemlich rein mathematisch und macht keinerlei physikalische (QFT, Renormierung) Analyse der Situation - nämlich der Kopplungskonstanten, Gegenterme oder Regulatoren, Symmetrien, die die Gesetze der Physik einschränken, falls vorhanden, und die Abhängigkeit oder Unabhängigkeit physikalischer Observablen von diesen Dinge. Also -1.

Obwohl ich nicht weiß, ob es einen allgemeinen Beweis gibt, denke ich, dass der Casimir-Effekt einer renormierbaren Quantenfeldtheorie vollständig durch eine Theorie der Renormierung auf Mannigfaltigkeiten mit Rand verstanden werden sollte. Das wesentliche Merkmal ist, dass man die Renormierung der Kopplungskonstanten in den Randtermen im Allgemeinen nicht vernachlässigen kann. Unter Verwendung dieser Strategie beobachteten Bondag und Vassilevich , dass die Renormierung des Oberflächenspannungsterms (die vollständige Aktion einschließlich der Oberflächenterme ist in Gleichung 44 angegeben) einen Gegenterm liefert, der einen divergierenden Term in der Casimir-Energie einer dielektrischen Kugel aufhebt, der nicht aufgehoben werden kann durch die Nullpunktenergiesubtraktion (wie von den gleichen Autoren zusammen mit Kirsten in einer früheren Arbeit hervorgehoben wurde). In der späteren Arbeit verifizierten die Autoren, dass das Modell einschließlich der Oberflächenterme in einer Schleife renormierbar ist.

Gute physikalische Antwort, +1, im Gegensatz zu einer billigen Produktion von URLs zu verehrten Fields-Medaillengewinnern, die trotz ihrer Größe so gut wie keinen Beitrag zur Antwort auf die ursprüngliche Frage leisten, nämlich was an den Parametern, die diese physikalischen Situationen beschreiben, einstellbar ist (Platten vs. andere Geometrien).
Man muss verstehen, dass jede Randbedingung (wie E = 0) eine Lösung für die gekoppelten Gleichungen von Ladungen und Feldern ist. Der Casimir-Effekt ist also eine Wechselwirkung von Ladungen in der QFT, nicht nur ein "Vakuumfeldeffekt".

(Dieses Argument gilt für ein eindimensionales System, aber ähnliche Argumente können in höheren Dimensionen angegeben werden. Wir arbeiten in Einheiten mit c = = e = 1 ).

Angenommen, wir haben eine Reglerprozedur, die durch eine Impulsabschaltung parametrisiert ist Λ . Dann auf Distanz L zwischen zwei parallelen Platten können wir die regularisierte Energiesumme in Potenzen des Cutoffs erweitern als

E = a L Λ 2 + b L Λ + c L + Ö ( 1 Λ ) .
Wir können die Abhängigkeit von ableiten a L und b L durch Dimensionsanalyse. Die ersten beiden Terme stammen aus dem Beitrag sehr hoher Energien zur Summe. Daher können sie beispielsweise nicht von Dingen wie der Elektronenmasse abhängen, die bei hohen Energien vernachlässigbar ist; Die einzige verfügbare Energieskala ist 1/L. Somit ist die einzige von der Dimensionsanalyse erlaubte Abhängigkeit a L = a L , b L = β für einige dimensionslose Konstanten a und β . Also erhalten wir
E = a L Λ 2 + β Λ + c L + Ö ( 1 Λ ) .
Es ist klar, dass der zweite Term wegfällt, wenn wir die Kraft berechnen d E / d L . Was ist mit dem ersten Semester? Beachten Sie, dass auf beiden Seiten jeder Platte immer ein Vakuum herrscht . Wenn wir eine Platte vorbeischieben d x , dann ergibt der erste Term eine Erhöhung der Energie des Vakuums links um a Λ 2 d x , aber es ergibt sich auch eine Verringerung der Energie des Vakuums rechts um den gleichen Betrag. Solange wir also die Ableitung der gesamten Vakuumenergie berechnen, fällt auch der erste Term weg.

Dies zeigt, dass (in der Grenze des großen Cutoffs, so dass die Ö ( 1 / Λ ) Begriffe verschwinden) kommt die Kraft allein aus c L , die tatsächlich unabhängig vom Cutoff ist! Man möchte hoffen, dass der Begriff tatsächlich unabhängig von allen Details der Regulierungsbehörde ist; Der Beitrag von Terence Tao (auf den bereits in Aarons Antwort verwiesen wird) beweist dies in einem Sonderfall, dh dem freien masselosen Boson.

Die gleiche Frage haben Sie sich vielleicht bei jeder Berechnung in der Quantenmechanik gestellt: Warum liefern unterschiedliche Regulatoren für formal divergierende Größen das gleiche Ergebnis? Die Antwort ist, dass unter der Annahme, dass diese Regler wichtige Symmetrien, die unsere Messungen beeinflussen (wie zum Beispiel die Lorentz-Invarianz), nicht brechen, die Regler nur die Physik im UV durcheinander bringen und die IR-Physik gleich lassen. Wenn wir uns für Niedrigenergiephysik interessieren, dann werden verschiedene Regler alle die Größen, an denen wir interessiert sind, unberührt lassen und nur irrelevante Hochenergiedetails modifizieren. Das ist kein Wunder! Siehe auch die Diskussion in: Welche Renormierungstechniken sind für 3+1 QED verfügbar?

In Casimirs Originalarbeit stellte er eine Ableitung vor, die zeigt, dass jede "vernünftige" Wahl des Reglers dasselbe Ergebnis liefern wird. Dies ist im Wesentlichen das, was Terry Tao in seinem Blogbeitrag durchführt, aber sein Beitrag ist ziemlich detailliert und vielleicht könnte eine besser verdauliche Präsentation nützlich sein! Es geht so:

In einer eindimensionalen Box werden die Frequenzen zu denen ebener Wellen quantisiert, ω n = π n wo ist die Größe der Kiste. Lassen Sie uns dann definieren

E ( ) = n 1 2 ω n
die Summe der Nullpunktsenergien dieser Moden. Modifizieren wir diese Summe nun mit einem (meistens) willkürlichen Regler,
E ( ) = π 2 n n   f   ( n Λ )
wobei wir einfach jeden Term in der Summe mit multipliziert haben f ( n Λ ) . Wir nehmen nur an, dass diese Funktion f ( x ) stirbt schneller ab als x 1 , und das f ( 0 ) = 1 . Die erste Anforderung ist einfach, dass der Regler nur bei hohen Energien Modi abschaltet - die UV-Frequenzen gehen direkt durch die Box. Die zweite Anforderung ist nur, dass der Regler das Spektrum im IR nicht beeinflussen sollte.

Stellen wir uns nun vor, wir hätten ein Paar Orte, die in einer Entfernung voneinander getrennt sind a , in einer größeren Schachtel der Größe L , unten gezeigt 1 :

Geben Sie hier die Bildbeschreibung ein

Wenn wir gehen L fest und variieren a , die Kraft auf die Wände unserer Platten in der Ferne a ist d E / d a . Die Energie der L a Seite der Box ist

E ( L a ) = π 2 n n ( L a )   f   ( n ( L a ) Λ )

und wenn wir die Kontinuumsgrenze nehmen L , wir bekommen

E ( L a ) π 2 ( L a ) Λ 2 x f ( x ) d x
Wenn wir also jetzt die diskrete Summe für zwischen den Platten zur Kontinuumssumme für die Außenseite der Platten addieren, erhalten wir
E t Ö t a l = E ( a ) + E ( L a ) = π 2 L Λ 2 x   f ( x ) d x + π 2 a ( n   n   f ( n a Λ )   n   f ( n a Λ ) d n )
Die Differenz zwischen der Summe einer Funktion und ihrem Integral ergibt sich aus der Euler-Maclaurin-Formel:
n = 1 N f ( n ) 0 N f ( n ) d n = f ( 0 ) + f ( N ) 2 + f ' ( 0 ) f ( 0 ) 12 + . . . B j f ( j 1 ) ( 0 ) + f ( j 1 ) ( N ) j !
wo B j sind die Bernoulli-Zahlen.

Okay, jetzt kümmern wir uns nur um die Teile dieses Ausdrucks, die davon abhängen a , da wir in Bezug auf differenzieren werden a . Isolieren Sie diese Begriffe,

E t Ö t a l = π f ( 0 ) 24 a B 4 4 ! 3 π 2 a 3 Λ 2 f ( 0 ) + . . .
bei dem die . . . Begriffe werden durch zunehmende Potenzen von unterdrückt Λ . Wenn Λ als groß angesehen wird, werfen wir alles außer dem ersten Term weg; unter Hinweis darauf, dass wir benötigt f ( 0 ) = 1 wir erhalten dann die Casimir-Kraft
F = d E d a = π 24 a 2

Nun ist es den Menschen oft ein Rätsel, warum zB esoterische Manipulationen wie die Zeta-Regularisierung die richtige Antwort geben. Das obige Argument entmystifiziert, warum gute Regler gute Regler sind: Sie können die Differenz zwischen der tatsächlichen Summe, die zu einem Cutoff summiert wird, und der Zeta-Summe (oder welcher Summe auch immer - Wärmekernsumme vielleicht) mit Euler-Maclaurin erweitern und zeigen, dass es eine Universalität gibt Stück, das von der Wahl des Abschnitts nicht berührt wird. Schlimmes kann nur passieren, wenn die Regulatoren Symmetrien brechen.

Sie befürchten, dass dies bei unterschiedlichen Geometrien oder Abmessungen nicht funktioniert - aber es funktioniert! Solange der Regler die Symmetrie der Geometrie nicht bricht, ist alles in Ordnung. Es ist nicht klar, was Ihr 2 d Das skalare Beispiel, an das Sie denken, ist, aber wenn es sich um ein Problem vom Typ Casimir handelt, hängt der unendliche Teil sicherlich nicht von der Plattentrennung ab, also geht es uns gut. Eine frühere und interessante Antwort weist darauf hin, dass der Regler manchmal mehr tun muss, als nur "die Nullpunktenergie abzuziehen", wenn die Platten eine seltsame Grenze haben. Unter dem Strich müssen Sie jedoch nur in gewisser Weise mit der UV-Physik herumspielen die die Symmetrien der Grenze respektiert.

1 Bild (und Ableitung) schamlos aus dem QFT-Lehrbuch von Matt Schwartz gestohlen.

Paar Plätze -> Paar Platten
Wie erklärt dies die Zeta-Regularisierung? | ω 0 | s ist nicht gleich 1!

Größen, die wir im Labor messen können, sind endlich. Wir haben mathematische Modelle, die bei naiver Anwendung unendliche Werte für messbare Größen liefern. Betrachten Sie diese Artefakte mit unendlichen Werten des spezifischen mathematischen Modells/Werkzeugs/der Berechnungsmethode. Wenn man Wege identifizieren kann, in denen eine bestimmte Art von Unendlichkeit in einem mathematischen Modell / einer mathematischen Methode konsistent auftritt, kann man sie verfolgen und auf konsistente Weise "subtrahieren, wenn sie ausgeschaltet ist" (dh sie renormieren).

Wenn für eine bestimmte Art des Rechnens Unendlichkeiten auf willkürliche Weise entstehen ... dann gibt es keine Möglichkeit, sie auf mathematisch konsistente Weise zu entfernen, und Sie hören nichts von dieser bestimmten Art und Weise.

Jedes Unendliche kann mathematisch reguliert werden :) Eine andere Frage ist, ob es vom Physiker zugelassen wird.